+1 Daumen
4,1k Aufrufe

Gesucht ist der Beweis durch vollständige Induktion.

\( H(n): n^{3} \leq 3^{n}, \quad \forall n \in \mathbb{N}, n \geq 4 \)

Avatar von

1 Antwort

0 Daumen
 
Beste Antwort

Hey ;)

Fälle für n=1,2,3 von Hand überprüfen. Induktionsschritt für n >= 4 ist dann:

$$3^{n+1} = 3^n \cdot 3 \overset{\text{I.V.}}{\underset{\text{}}{ \ge }} n^3 \cdot 3 = n^3 + n^3 + n^3 $$

$$\ge n^3 + 3n^2 + 3n^2 \ge n^3 + 3n^2 + 3(2n) \ge n^3 + 3n^2 + 3(n+1)$$

$$= n^3 + 3n^2 + 3n + 3 \ge n^3 + 3n^2 + 3n + 1 = (n+1)^3 \ .$$

Dabei wurden folgende offensichtliche Ungleichungen verwendet:

$$ n^3 \ge 2n^2 \ , \quad für \quad n \ge 4$$

$$ n^2 \ge 2n \ge n+1 \ , \quad für \quad n \ge 4$$

 

Ich hoffe die offensichtlichen Ungleichungen sind klar? Bei der ersten einfach beide Seiten durch n2 teilen und bei der zweiten erst durch n teilen und dann minus n rechnen.

Avatar von 1,6 k

Ein anderes Problem?

Stell deine Frage

Willkommen bei der Mathelounge! Stell deine Frage einfach und kostenlos

x
Made by a lovely community